Jump to content

A difficult problem


glyphomouni

Recommended Posts

I was trying to prove the following inequality and really got stuck

 

The inequality is. [math]\frac{x^4+y^4+z^4}{x^2y^2+x^2z^2+y^2z^2}\geq\frac{2m+3n}{3(m+n)}[/math] for all the reals x,y,z different to zero and for all the natural Nos m,n

 

Can we use induction here

Link to comment
Share on other sites

This post belongs to homework help, and according to forum rules i am not allowed to give you answer. I can only give you hints.

you can express the numerator in terms of [imath](a+b+c)^2[/imath] formula. [imath](a+b+c)^2= a^2+b^2+c^2+2ab+2bc+2ac[/imath].

 

it'l be [imath](a+b+c)^2-2ab-2bc-2ac[/imath]

[imath](a+b+c)^2-2(ab+bc+ac)[/imath]

[imath]=\frac{(a+b+c)^2}{ab+bc+ac}-2[/imath]

 

hey i think you will get L.H.S=0 if you multiply 1/-2 on both sides

Edited by Physicsfan
Link to comment
Share on other sites

i think it is possible to get the LHS as a square and the RHS as a negative number
Definitely not. Look at the OP. m and n are to be natural numbers, both sides are strictly positive.

Mr Skeptic is on the right track.

Link to comment
Share on other sites

Thank you all ,but could you be more specific?

 

R.H.S is a Natural No ,but what manipulations can make the ratio:[math]\frac{x^4+y^4+z^4}{x^2y^2+x^2z^2+y^2z^2}[/math] such a complicated Natural No??.

 

Why not a whole Natural No??

 

I have to hand in that exercise next Monday and i am still in the dark.

 

Thanx

Link to comment
Share on other sites

The LHS isn't a natural number (nor is the RHS), but really that's not the point.

 

Basically, you know that x2,y2,z2,m and n are all strictly positive - that's all you should need.

 

If you can prove that

x2x2+y2y2+z2z2> x2y2 + y2z2 + z2x2

Then you will know that LHS>1

 

If you can prove that 2m + 3n < 3m + 3n

Then you will know that RHS< 1

 

Since LHS>1>RHS it follows that LHS>RHS.

Link to comment
Share on other sites

Thanks .

 

In that case because 2m<3m ,then 2m+3n<3(m+n) or [math]\frac{2m+3n}{3(m+n)}<1\leq\frac{x^4+y^4+z^4}{x^2y^2+x^2z^2+y^2z^2}[/math].

 

I am worried about the equality sign, because [math]A<1\leq B[/math] =>A<B.

 

Is it easy to prove [math]\frac{x^4+y^4+z^4}{x^2y^2+x^2z^2+y^2z^2}\geq 1[/math]??

Link to comment
Share on other sites

Is it easy to prove [math]\frac{x^4+y^4+z^4}{x^2y^2+x^2z^2+y^2z^2}\geq 1[/math]??
It's not easy, no, but I managed it in a few lines by taking a geometric view.

 

This does suppose a basic knowledge of dot products, which I'm going to assume because it makes this proof nice and very, very general. (which is always good).

 

Suppose we have two vectors of equal length in one vector space, |v|=|w|.

 

Now it's trivial to prove that:

v.v > v.w

(use the cosine definition of dot products)

 

Now, assign those vectors:

v=(x2,y2,z2)

w=(y2,z2,x2)

 

You'll then have to show that

 

|v|=|w|

 

v.v=x2x2+y2y2+z2z2

 

v.w=x2y2+y2z2+z2x2

 

And then you'll have that

 

x2x2+y2y2+z2z2> x2y2 + y2z2 + z2x2

 

Which should be all you need.

Link to comment
Share on other sites

Before i learn your proof ,there is a thing that i am worried about.

 

if we prove [math]\frac{x^4+y^4+z^4}{x^2y^2+x^2z^2+y^2z^2}\geq 1>\frac{3m+2n}{3(m+n)}[/math] that implies [math]\frac{x^4+y^4+z^4}{x^2y^2+x^2z^2+y^2z^2}>\frac{3m+2n}{3(m+n)}[/math] and not

 

 

[math]\frac{x^4+y^4+z^4}{x^2y^2+x^2z^2+y^2z^2}\geq\frac{3m+2n}{3(m+n)}[/math] .

 

Is that right??

Link to comment
Share on other sites

Yes, it is overkill but [ [A>B] => [A> B] ] is so trivial that you shouldn't even need to state it - just state what you've proven, and the conclusion that it implies.

 

(I'm not sure why you'd have been asked to prove that when a slightly more powerful statement is provable, but there you go)

Link to comment
Share on other sites

Another method is to use the Cauchy–Schwarz inequality:

 

[math]\color{white}......[/math][math]x^2y^2+y^2z^2+z^2x^2[/math]

 

[math]\leqslant\ \left(x^4+y^4+z^4\right)^{\frac12}\left(y^4+z^4+x^4\right)^{\frac12}[/math]

 

[math]=\ x^4+y^4+z^4[/math]

 

[math]\therefore\ \frac{x^4+y^4+z^4}{x^2y^2+y^2z^2+z^2x^2}\ \geqslant\ 1\quad\mbox{if}\quad xyz\ne0[/math]

Link to comment
Share on other sites

Thank you .

 

But the frame within which we work is high school level. Thus although we know a little bit of vectors we have not done the Cauchy–Schwarz inequality.

 

Anyway thanks for the reference i will look it up

Link to comment
Share on other sites

Create an account or sign in to comment

You need to be a member in order to leave a comment

Create an account

Sign up for a new account in our community. It's easy!

Register a new account

Sign in

Already have an account? Sign in here.

Sign In Now
×
×
  • Create New...

Important Information

We have placed cookies on your device to help make this website better. You can adjust your cookie settings, otherwise we'll assume you're okay to continue.